Block 2 Flashcards
A 67 year old man presents to the emergency department with shaking chills and a temperature of 101F. Laboratory examination reveals a hematocrit of 23%, and urine tests are positive for blood. The patient states that he is taking only one medication for his irregular heartbeat. Which of the following drugs most likely caused the appearance for these symptoms in this patient?
A. Verapamil B. Digoxin C. Propanolol D. Quinidine E. Hydralazine
D. Quinidine (with hemolytic anemia)
An elderly man presents with complaints of ringing in his ears, blurred vision, and upset stomach. He is taking multiple medications. His wife states that he has few episodes of confused, delirious behavior over the past few weeks. Which of the following agents might be responsible for this man’s symptons?
A. Allopurinol B. Hydralazine C. Spironolactone D. Quinidine E. Niacin
D. Quinidine
CNS symptoms
Figure below illustrates a current concept of the control of gastric acid secretion. Which of the following drugs acts at the site labeled "ATPase"? A. aluminum hydroxide B. misoprostol C. omeprazole D. ranitidine E. sucralfate
C. Omeprazole
Proton pump inhibitors
A 35 year old woman appears to have familial combined hyperlipidemia. Her serum concentrations of total cholesterol, LDL cholesterol, & triglyceride are elevated. Her serum concentration of HDL cholesterol is somewhat reduced. Which of the following drugs is most likely to cause an increase in this patient’s TG and VLDL cholesterol when used as monotherapy?
A. Atorvastatin B. Gemfibrozil C. Cholestyramine D. Niacin E. All of the above
C. Cholestyramine
Bile binding
A 52 year old man with peptic ulcer disease has been on drug therapy for 3 months and has noticed changes in his bowel habits, increasing headaches, dizziness, skin rashes, loss of libido and gynecomastia. Which of the following drugs is most likely responsible for these side effects?
A. Cimetidine B. Metronidazole C. Sucralfate D. Omeprazole E. Ranitidine
A. Cimetidine
A 58 year old business executive is brought to the emergency room 2 hours after the onset of severe chest pain during a vigorous tennis game. She has a history of poorly controlled mild hypertension and elevated blood cholesterol but does not smoke. ECG changes cnfirm the diagnosis of myocardial infaraction. The decision is made to attempt to open her occluded artery. Conversion of plasminogen to plasmin is brought about by
a. Heparin
b. Warfarin
c. Reteplase
d. Aminocaproic acid
e. Lepirudin
c. Reteplase
Only tPa
A patient with multiple medical problems is taking several drugs, including theophylline, warfarin, quinidine, and phenytoin. Despite the likelihood of interactions, dosages of each are adjusted carefully so their serum concentrations and effects are acceptable. However, the patient suffers some GI distress and starts taking a drug provided by one of his “well intentioned” friends. He presents with excessive or toxic effects from all his other medications and blood tests reveal that their serum concentrations are high. Which was the drug the patient most likely self-prescribed and took?
A. Nizatidine B. Sucralfate C. Ranitidine D. Famotidine E. Cimetidine
E. Cimetidine
***A 62 year old white man complains of left thigh and leg pain and swelling that are exacerbated by walking. One week earlier, the patient underwent cardiac catheterization. The patient is currently vacationing and has spent the past 28 hours in a car. Which of the following drugs, which might be prescribed in this instance, works by inhibiting the enzyme epoxide reductase?
A. Tissue-type plasminogen activator (tPa) B. Dipyridamole C. Heparin D. Streptokinase E. Acetylsalicylic acid F. Warfarin
F. Warfarin
Warfarin affects INR in blood
***A 67 year old woman with Parkinson disease responds well to treatment with a combination of levodopa and carbidopa. The woman develops gastroesophageal reflux disease (GERD) and is prescribed an appropriate drug for its management. She is instructed to continue talking levodopa and carbidopa.. After she starts taking the drug for GERD, she complains that her symptoms of Parkinson disease have gotten worse. The drug most likely prescribed to treat GERD in this patient was
A. famotidine B. bethanechol C. cisapride D. metoclopramide E. atropine
D. metoclopramide
If a patient undergoes a percutaneous coronary procedure and placement of a stent in a coronary blood vessel, she may be given EPTIFIBATIDE. The mechanism of EPTIFIBATIDE’s anticlotting action is
A. Reversible inhibition of glycoprotein IIb/IIIa receptors
B. Blockade of posttranslational modification of clotting factors
C. Inhibition of thromboxane production
D. Activation of antithrombin III
E. Irreversible inhibition of platelet ADP receptors
A. Reversible inhibition of glycoprotein IIb/IIIa receptors
A 58 year old woman is being treated for chronic suppression of a ventricular arrhythmia. After two months of therapy, she complains about feeling tired all the time. Examination reveals a resting heart rate of ten beats per minute lower than her previous rate. Her skin is cool and clammy. Laboratory test results indicate low thyroxin and elevated thyroid-stimulating hormone levels. Which of the following antiarrhythmic drugs is the likely cause of these signs and symptoms?
A. Amiodarone B. Quinidine C. Propanolol D. Verapamil E. Procainamide
A. Amiodarone
A 64 year old man presents to his physician with aching, burning pain after meals. He has been self medicating with antacids for several months but has found this to be increasingly ineffective. His physician decides to take him off the antacids and instead places him on a combination of Ranitidine and sucralfate. Why is this combination a bad idea?
A. Ranitidine increases the toxicity of sulcralfate
B. Ranitidine inhibits the action of sucralfate
C. Sucralfate and ranitidine co precipitate
D. Sucralfate increases the toxicity of ranitidine
E. Sulcralfate inhibits the actions of Ranitidine
B. Ranitidine inhibits the action of sucralfate
Which of the following drugs reduces LDL cholesterol by inhibiting an intestinal transport protein?
A. Atorvastatin B. Cholestyramine C. Gemfibrozil D. Ezetimibe E. Niacin
D. Ezetimibe
A 25 year old woman comes to her family physician for a routine check-up. Her physical examination shows a mildly overweight woman but is otherwise unremarkable. A fasting lipid panel, however, shows an LDL cholesterol level of 310 mg/dL, HDL cholesterol level of 42 mg/dL, triglyceride level of 150 mg/dL, and total cholesterol level of 382 mg/dL. Because a diagnosis of familial hpercholesterolemia is suspected, the doctor initiates treatment of her condition. Soon after starting treatment, however, she presents with myalgias. Laboratory values show elevated levels of aspartate aminotransferase, alanine aminotransferase, and creatinine kinase. Which of the following interventions is most likely responsible for the patient's myalgias? A. B complex vitamins B. hormone replacement therapy C. Liver transplant D. Low HDL E. Statin Medications
E. Statin Medications
A 35 year old woman appears to have familial combined hyperlipidemia. Her serum concentrations of total cholesterol, LDL cholesterol, and triglyceride are elevated. Her serum concentration of HDL cholesterol is somewhat reduced. If this patient is pregnant, which of the following drugs should be avoided because of a risk of harming the fetus? A. Cholestyramine B. Cholestipol C. Pravastatin D. Niacin E. Vitamin B3
C. Pravastatin
NO STATINS IN PREGNANCY
A 20 year old varsity hockey player is referred to you by his coach. The young athlete has excessive bruising after a very physical match 2 days before. His knee had been bothering him, so he took two 325-mg aspirin tablets several hours before the contest. He got checked hard into the boards. 10 times during the game, but denies any excessive or unusual trauma. As you ponder the etiology you order several blood tests. Which test or finding do you most likely expect to be abnormal as a result of the prior aspirin use?
A. Platelet count B. Bleeding time C. INR (International normalized ratio) D. Activated partial thromboplastin time (APTT) E. Prothrombin time
B. Bleeding time (prolonged)
***A patient with a supraventricular tachycardia has an atrial rate of of 280/min with a ventricular rate of 140/min via a 2:1 AV nodal transmission. After treatment with a drug, the atrial rate slowed to 180/min, but the ventricular rate increased to 180/min. Which of the following drugs was most likely to have been given to this patient?
A. Adenosine B. Quinidine C. Digoxin D. Esmolol E. Verapamil
B. Quinidine (increase in AV conduction)
A patient develops severe thrombocytopenia in response to treatment with unfractionated heparin and still requires parenteral anti coagulation. The patient is most likely to be treated with
A. Abciximab B. Lepuridin C. Streptokinase D. Urokinase E. Plasminogen
B. Lepuridin
***A 22 year old woman who experienced pain and swilling in her right leg presented at the emergency room. An ultrasound study showed thrombosis in the popliteal vein. The patient, who was in her second trimester of pregnancy, was treated for seven days with intravenous unfractionated heparin. The pain resolved during the course of therapy, and the patient was discharged on day eight. Which one of the following drugs would be most appropriate outpatient follow-up therapy for this patient, who lives 100 miles from the nearest hospital?
A. Streptokinase B. Warfarin C. LMWH D. Unfractionated heparin E. Alteplase
C. LMWH
Low molecular weight heparin
Cardiac output is a function of stroke volume and heart rate. Stroke volume increases when contractility increases, preload increases, or afterload decreases. There are a number of factors that affect each of these components and ultimately cardiac output. Which of the following variations woul increase cardiac output?
A. Atenolol treatment B. Verapamil treatment C. Clonidine D. Adenosine E. Cardiac Glycoside administration
E. Cardiac Glycoside administration
Digitalis group
A 67 year old woman is brought to the emergency department with complaints of fever, malaise and the recent appearance of a malar “butterfly” rash on the face. The patient states that she is taking one medication for arrhythmias. If she is positive for antihistone and is mildly anemic, which of the following medications is she most likely taking?
A. Mexiletine B. Disopyramide C. Flecainide D. Procainamide E. Digoxin
D. Procainamide
A 29 year old man has had episodes of watery diarrhea and abdominal pain for the past year. He has been defecating as many as 10 times a day during the past week, nd he has lost about 8 kg (18 lb) during the past 3 months. His temperature is 38.3 degrees C (101 F). Laboratory studies and sigmoidoscopy findings are consistent with the diagnosis of severe acute ulcerative colitis. Which of the following drugs would be most appropriate to administer as an enema for short-term treatment of acute exacerbations of ulcerative colitis in this patient?
A. Infliximad B. Cyclosporine C. Amoxicillin D. Hydrocortisone E. Sulfasalazine
D. Hydrocortisone
***Which one of the following drugs causes a decrease in liver triacylglycerol synthesis by limiting available free fatty acids needed as building blocks for this pathway?
A. Probucol B. Niacin C. Cholestyramine D. Lovastatin E. Fenofibrate
B. Niacin or Nicotinic acid
A 67 year old woman with a history of venous thromboembolism is placed on warfarin (Coumadin) prophylactically. The blood concentration of Coumadin becomes too high and bleeding occurs. This bleeding can best be treated by the administration of which of the following?
A. Fibrinogen B. Thrombin C. Platelets D. Protein C E. Vitamin K
E. Vitamin K
A 41 year old woman who is diabetic presents to her physician complaining of GI distress and bloating particularly after meals. Which of the following drugs is useful to her?
A. Sucralfate B. Metoclopramide C. Famotidine D. Omeprazole E. Ranitidine
B. Metoclopramide
If quinidine and digoxin are administered concurrently, which of the following effects does quinidine have on digoxin?
A. The absorption of digoxin form the GI tract is decreased
B. The effect of digoxin on the AV node is antagonized
C. The concentration of digoxin in the plasma is increased
D. The metabolism of digoxin is prevented
E. The ability of digoxin to inhibit the Na+K+ stimulated ATPase is reduced
C. The concentration of digoxin in the plasma is increased
A 44 year old obese man has extremely high plasma triglyceride levels, but cholesterol levels are within normal limits. Following treatment with a drug specifically indicated for hypertriglyceridemia, triglyceride levels decrease to almost normal. Which of the following agents is most likely to have caused this desired change?
A. Atorvastatin B. Cholestyramine C. Ezitemibe D. Colestipol E. Gemfibrozil
E. Gemfibrozil
Gemfibrozil - “fibrate” - clofibrate, ferofibrate +LPL Decrease TG
A patient in the coronary care unit develops episodes if paroxysmal AV nodal reentrant tachycardia (PSVT). Which of the following would generally be considered a first-line drug for promptly stopping the arrhythmia?
A. Adenosine B. Edrophonium C. Digoxin D. Phenylephrine E. Propranolol
A. Adenosine
A couple celebrating their fortieth wedding anniversary is given a trip to Peru to visit Machu Picchu. Due to past experiences while traveling, they ask their doctor to prescribe an agent for diarrhea. Which of the following would be effective?
A. Lorazepam B. Omeprazole C. Famotidine D. Loperamide E. All of the above
D. Loperamide
A patient who has been a high-dose alcohol abuser for many years presents with hepatic portal-stsemic encephalopathy. Which of the following drugs, give in relatively high doses, would be most suitable for the relief of signs and symptoms of this condition, and the likely underlying biochemical anomalies?
A. Lactulose B. Esomeprazole C. Diphenoxylate D. Ondansetron E. Loperamide
A. Lactulose
Binds to ammonia ion and excretes it
***Following a myocardial infaraction, a patient in the emergency room of a hospital develops ventricular tachycardia. The best way to manage this situation is with the administration of
A. Dilitazem B. Flecainide C. Amiodorone D. Lidocaine E. Adenosine
D. Lidocaine
A patient is admitted for acute coronary syndrome with signs of impending myocardial infarction. She undergoes angioplasty with double coronary stenting to maintain the patency of her coronary vessels. Which of the following drugs will probably be administered to prevent clotting in the region of the wire mesh stents?
A. Dilitazem B. Flecainide C. Amiodorone D. Lidocaine E. Adenosine
??
Clopidogrel
The primary process in the formation of this arterial clot is platelet aggregation, so an antiplatelet drug such as clopidogrel is suitable
or Lidocaine (if it is the option)
A 21 year old man is brought by his roommate to the emergency department because of abrupt onset of shortness of breath, mild chest pain and sensation of rapid heart beating. ECG documents. Supraventricular arrhythmias wiht pulse of 200 per minute. Under ECG monitoring gentle massage over right carotid sinus is attempted but attack doesn’t cease. Which of the following is most appropriate next step of managaement?
A. IV Lignocaine B. Further carotid massage C. IV Procainamide D. IV Verapamil E. IV Amlodipine
D. IV Verapamil (or diltiazem)
ABCD - Calcium Channel Blocker
A 68 year old woman with a history of coronary insufficiency suffers a heart attack and begins taking anticoagulant. One year later, her physician prescribes colestyramine to treat her hypercholesterolemia. A few months after the patient begins colestyramine treatment, she develops gastrointestinal bleeding and is found to have hypothrombinemia. Which of the following mechanisms accounts for how colestyramine causes hypoprothrombinemia?
A. Distribution B. Absorption C. competes with plasma proteins D. Metabolism E. Pharmacodynamics
B. Absorption
***A 63 year old woman develops tachyarrhythmia after recovering from an MI. Her physician prescribes a new medication. Five montgs later she presents to the clinic complaining of difficulty breathing and dry coughing. Neither of which she has experienced previously. Physical examination reveals decreased breath sounds. After ruling out cardiac causes for her symptoms, her physician conducts a series of tests. X ray of the chest shows decreased lung volume. Lung function tests show decreased vital capacity. Which agent most likely caused her current condition?
A. Amiodarone B. Procainamide C. Quinidine D. Verapamil E. Proponolol
A. Amiodarone
Restrictive lung disease
Amiodarone - thyroid dysfunction, causes pulmonary fibrosis and smurf skn.
A patient with peptic ulcer disease is taken off their medication because of undesirable side effects. As a result, the patient has rebound gastric acid hypersecretion. Which of the following drugs best accounts for the observed result?
a. An H 1 -receptor antagonist
b. A proton pump inhibitor
c. A cholinergic receptor antagonist
d. An antacid
e. A CCKB receptor antagonist
D. An antacid
acid rebound
- **We have a 28-year-old female patient with Stage II essential hypertension, tachycardia, and occasional palpitations (ventricular ectopic beats). Normally we might consider prescribing a beta blocker to control the blood pressure and cardiac responses, but our patient also has asthma, and she is trying to get pregnant. Which of the following drugs would be the best alternative to the beta blocker in terms of likely efficacy on pressure and heart rate, and in terms of relative safety?
a. Diltiazem
b. Enalapril
c. Furosemide
d. Phentolamine
e. Prazosin
A. Diltiazem(calcium channel blocker)
Asthma, no non selective B Blocker
***An elderly male patient who has just been referred to your practice has been taking a drug for symptomatic relief of benign prostatic hypertrophy.
In addition to its effects on smooth muscles of the prostate and urethra, this drug can lower blood pressure in such a way that it reflexly triggers tachycardia, positive inotropy, and increased AV nodal conduction. The drug neither dilates nor constricts the bronchi. It causes the pupils of the eyes to constrict and interferes with mydriasis in dim light. Initial oral dosages of this drug have been associated with a high incidence of syncope. Which prototype is most similar to this unnamed drug in terms of the pharmacologic profile?
a. Captopril
b. Hydrochlorothiazide (prototype thiazide diuretic)
c. Labetalol
d. Nifedipine
e. Prazosin
f. Propranolol
g. Verapamil
E. Prazosin
Alpha 1 beta blockers - BPH (Benign prostatic hypertrophy
***We have just diagnosed Stage 1 essential hypertension in a 30-year old
man who has a history of asthma. He regularly uses an inhaled corticosteroid,
which seems to work well, but does need to use an albuterol inhaler about once every 3 weeks for suppression of asthma attacks. Which antihypertensive drug or drug class poses the greatest risk of exacerbating the patient’s asthma and counteracting the desired pulmonary effects of the albuterol, even though it might control his blood pressure well?
a. Diltiazem
b. Hydrochlorothiazide
c. Labetalol
d. Ramipril
e. Verapamil
c. Labetalol (increases asthmatic affects, non selective Beta Blocker )
Asthma, stage 1 HTN,
A 56 year old woman with a long history of untreated hypertension brought to emergency department because of severe headache, confusion. There is disorientation to person but not to time and place. BP is 230/180, Pulse 86/min, respiratory rate is 18/min. Fundoscopic examination reveals optic disc edema and dipstick test shows protein in urine. Which of the following is most appropriate pharmacotherapy?
A. ACEI B. Proponolol C. Nifedipine D. Sodium nitroprusside E. Furosemide
D. Sodium Nitroprusside
Protein in urine, malignant hypertension
A diabetic male presents with hypertension and 24-h urine showing 200 mg of albumin. In a diabetic patient with microalbuminuria, the approproate drug for treatment of hypertension to prevent progression of renal failure is
A. Beta blocker B. Thiazide diuretic C. Enalapril D. Short-acting dihydropyridine calcium channel blocker for precise control (nifedipine) E. Sodium Nitroprusside
C. Enalapril (angiotensin converting enzyme inhibitors
Diabetes with HTN
A 52 year old man with a 30 pack year history of cigarette smoking presents to a physician. He was told that he has high cholesterol about 2 years ago, and he has history of mild HTN for which he has never been treated. He had MI 6 months ago. His post MI has been uncomplicated. His medications include one aspirin tablet every other day. Physical examination is normal except 4th heart sound. Which of the following is most appropriate next step in management to prevent morbidity and mortality?
A. Add beta blocker B. Add diuretics C. Add calcium channel blocker D. Increase aspirin dose three times daily E. Prescribe nitroglycerin for angina
A. Add beta blocker
*Morbidity and mortality
- **We have a patient with newly diagnosed essential hypertension, and start them on a commonly used antihypertensive drug at a dose that is considered to be therapeutic for the vast majority of patients. Soon after starting therapy the patient experiences crushing chest discomfort. EKG changes show myocardial ischemia. Studies in the cardiac cath lab show episodes of coronary vasospasm, and it is likely the antihypertensive drug provoked the vasoconstriction. Which of the following antihypertensive drugs or drug class most likely caused the ischemia and the angina?
a. Atenolol
b. Diltiazem
c. Hydrochlorothiazide
d. Losartan
e. Metolazone
A. Atenolol or Proponolol
(No beta blockers in vasospastic angina)
Anti HTN drug cause ischemia and angina - non selective B blocker (exacerbate vasoconstriction)
***A patient is hospitalized and waiting for coronary angiography. His history includes angina pectoris that is brought on by “modest” exercise, and is accompanied by transient electrocardiographic changes consistent with myocardial ischemia. There is no evidence of coronary vasospasm. In
the hospital he is receiving nitroglycerin and morphine (slow intravenous infusions), plus oxygen via nasal cannula.
He suddenly develops episodes of chest discomfort. Heart rate during these episodes rises to 170-190 beats/min; blood pressure reaches 180-200/110-120 mm Hg, and prominent findings on the EKG are runs of ventricular ectopic beats that terminate spontaneously, plus ST-segment elevation.
Although there are several things that need to be done for immediate care, administration of which one of the following is most likely to remedy (at least temporarily) the majority of these signs and symptoms and pose the lowest risk of doing further harm?
a. Aspirin
b. Captopril
c. Furosemide
d. Labetalol
e. Lidocaine
f. Nitroglycerin (increased dose as a bolus)
g. Prazosin
C. Labetalol (decrease HR and decrease BP because it has beta and alpha blocking properties)
Given its combination of both α/β blocking effect, it is the best approach for managing the hypertension, the tachycardia, oxygen supply-demand imbalance that leads to both CP and the ischemic ST-changes, and the ventricular ectopy - which is probably a reflection of excessive catecholamine stimulation of β1
receptors
A hypertensive patient has been on long-term therapy with lisinopril for hypertension. The drug isn’t controlling pressure as well as wanted, so the physician decides to add Spirinolactone as the (only) second drug. Which of the following is the most likely outcome of adding this diuretic to the ACE inhibitor regimen?
A. Blood pressure would rise abruptly
B. Better BP control, but with a risk of hyperkalemia
C. Cardiac depression, because both drugs directly depress the heart
D. Cough that may be severe, even though there was no cough with lisinopril alone
E. Hypernatremia, because ACE inhibitors counteract triamterene’s natriuretic effect
B. Better BP control, but with a risk of hyperkalemia
A 55 year old male patient comes to his primary care physician for his routine health care examination. He is completely asymptomatic and has nothing significant in his past medical history. He has 10 pack years of cigarette smoking but he does not drink. His PR:80/min; BP: 150/78 mm Hg; Temperature: 37.1C(98.8F). Labs show hematocrit of 43%, blood glucose of 90 mg/dl, serum creatinine of 0.7 mg/dL, serum Na of 140 mEq/L and serum K of 4 mEq/L. Urinalysis and EKG are unremarkable. His blood pressure readings on subsequent visits are 155/80 mmHg, 160/78 mmHg and 150/70 mmHg. Life-style modifications fail to contrl his blood pressure. Which of the following is the best initial pharmacological therapy for the control of his blood pressure?
A. Thiazide diuretics B. Hydralazine C. Sodium Nitroprusside D. Angiotensin Receptor Blockers E. Sodium channel blockers
A. Thiazide Diuretics
A 55 year old African American female presents to the ER with lethargy and blood pressure of 250/150. Her family members indicate that she was complaining of severe headache and visual disturbance earlier in the day. They report a history severe asthma but no known kidney disease. On physical exam, papilledena and retinal hemorrhages are present. The best approach is
A. Intravenous labetalol therapy
B. Continuous-infusion nitroprusside
C. Clonidine by mouth to lower blood pressure slowly but surely
D. Diltiazem sublingually to lower blood pressure rapidly and remove the and remove the patient from danger
E. Further history about recent home antihypertensives before deciding current
therapy
b. Continuous-infusion nitroprusside
Malignant HTN - nitroprusside or labetolol (not with asthma)
A patient presents for treatment of his severe essential HTN. He is being treated with numerous medications for high blood pressure, and hydralazine was recently added to his medication regimen. He explains that he has been experiencing flushing and headaches since his last visit, when hydralazine therapy was started. Which of the following is an adverse effect of hydralazine?
A. Anginal attack B. Bradycardia C. First-dose orthostatic hypotension E. Nephrotoxicity F. Pulmonary embolism
A. Anginal attack, orthostatic HTN if given
Adverse effect of hydralazine
***A 48 year old hypertensive patient has been sucessfully treated with a thiazide diuretic for the last five years. Over the last three months, his diastolic pressure has steadily increased, and he has been started on an additional hypertensive medication. He complains of several instances of being unable to achieve an erection and that he is no longer able to complete three sets of tennis. The second anyhypertensive medication is most likelt which one of the following?
A. Captopril B. Losartan C. Minoxidil D. Metoprolol E. Nifedipine
D. Metoprolol
Beta blockers - decrease in sexual dysfunction
A patient with a history of hypertension, heart failure, and peripheral vascular disease has been on oral therapy with drugs suitable for each for about 3 months. He runs out of the medication and plans to have the prescriptions refilled in a week or so.
Within a day or two after stopping his medications he experiences an episode of severe tachycardia accompanied by tachyarrhythmias, and an abrupt rise of blood pressure to 240/140 mm Hg—well above pretreatment levels. He complains of chest pain, anxiety, and a pounding headache. Soon thereafter he suffers a hemorrhagic stroke.
Which of the following drugs or drug groups, the man suddenly stops taking, most likely causes these responses?
a. ACE inhibitors
b. Clonidine
c. Digoxin
d. Furosemide
e. Nifedipine (a long-acting formulation)
f. Warfarin
b. Clonidine
Rebound HTN - Hemorrhagic stroke if suddenly stopping medication
A 32 year old woman with pheochromocytoma is being treated with phenoxybenzamine. After surgical excision of the tumor, the patient has an episode of hypotension requiring 30 seconds of cardiopulmonar resusciation and subsequen treatment in the intensive care unit. The attending physician asks his intern what physiologic responses during resuscitation. What would have been observed following administration of epinephrine?
A. Decrease in blood pressure B. Decrease in heart rare C. Increase in blood pressure D. Increase in respiratory rate E. No change in vital signs
A. Decrease in Blood Pressure (Epinephrine reversal phenomenon)
Pheochromocytoma - alpha blocker
1st administer Epinephrine -> increase BP
Alpha Blocker X -> Decrease BP
2nd administer Epinephrine -> Decease in BP (Beta 2 activation)
What type of drug may cause a precipitous fall in blood pressure and fainting on the initial administration.
Prazosin
Which drug produces headaches, palpitations, flushing and peripheral edema as side effects?
Dihydropyridine calcium channel blockers
Which antihypertensive drugs can precipitate hypertensive crisis following abrupt cessation of therapy?
Clonidine
The 45-year-old man has recently been diagnosed with hypertension and started on monotherapy designed to reduce peripheral resistance and prevent sodium and water retention. He has developed a persistent cough. Which drug is most likely responsible for this side effect?
Losartan
Which of the following beta blockers reduce after load?
A) Metoprolol
B) Propranolol
C) Carvedilol
D) Atenolol
C. Carvedilol
What is the most likely mechanism by which losartan lowers blood pressure?
Prevents the cardiovascular effects of angiotensin II mediated by AT-1 receptors
Which drug is the first line treatment for an African-American patient with hypertension and diabetic nephropathy?
Enalapril
We treat a patient with a drug that affects the clotting-thrombolytic systems for a time sufficient to let the drug’s effects and blood levels stabi- lize at a therapeutic level. We then isolate platelets from a blood sample and test their in vitro aggregatory responses to ADP, collagen, PAF, and throm- boxane A2. Aggregatory responses to ADP are inhibited; responses to the other platelet proaggregatory agonists are unaffected. Which drug did we most likely administer to this patient?
a. Aspirin
b. Bivalirudin
c. Clopidogrel
d. Heparin
e. Warfarin
c. Clopidogrel
You are reviewing the medication history of a 59-year-old man. He has been taking ramipril, pravastatin, and metformin for the last 5 years; and escitalopram for the last 12 months to help manage his depression. At his last clinic visit, a year ago, he was told to continue his current medica- tions but also started on slow-release niacin because diet, exercise, and other lifestyle modifications, and his current medications, were not ade- quate. What was the most likely reason for adding the niacin?
a. Counteract deficiencies of B-vitamin absorption caused by the antidepressant
b. Counteract polyphagia, and overeating, caused by the metformin
c. Lower triglyceride levels that did not respond adequately to the statin
d. Prevent statin-induced neuropathy
e. Slow the progression of diabetic nephropathy caused by the ACE inhibitor
c. Lower triglyceride levels that did not respond adequately to the statin
A patient presents with severe hypertension and tachycardia. Blood chemistry results, MRI findings, and the overall clinical presentation point to pheochromocytoma. The tumor appears operable, but the patient will have to wait a couple of months for the surgery. We prescribe phenoxy- benzamine in the interim, with the goal of suppressing some of the major signs and symptoms caused by the tumor. Which of the following best summarizes what phenoxybenzamine does, or how it acts?
a. Controls blood pressure by blocking α-adrenergic receptors in the peripheral vasculature
b. Controls heart rate by selectively blocking β1-adrenergic receptors
c. Inhibits catecholamine synthesis in the adrenal (suprarenal) medulla
d. Lowers blood pressure by inhibiting angiotensin converting enzyme and bradykininase
e. Stimulates catechol-O-methyltransferase, thereby facilitating epinephrine’s metabolic inactivation
a. Controls blood pressure by blocking α-adrenergic receptors in the peripheral vasculature
- **We administer a drug with the intent of lowering a patient’s elevated LDL and total cholesterol levels, and raising HDL levels. The drug we choose inhibits cholesterol synthesis by inhibiting 3-hydroxy-3-methylglutaryl- coenzyme A (better known as [HMG CoA] reductase). Which of the fol- lowing drugs best fits this description and works by the stated mechanism of action?
a. Clofibrate
b. Gemfibrozil
c. Lovastatin
d. Nicotinic acid (niacin)
e. Probucol
c. Lovastatin
- **We are administering nitroprusside intravenously for control of severe hypertension during surgery. The dose has gotten too high, and the drug has been administered too long. Refractoriness to the antihyperten- sive effects has occurred. Blood pressure is rising, and other signs and symptoms of potentially severe toxicity develop. What is the main nitro- prusside metabolite that accounts for these problems?
a. A highly efficacious α-adrenergic agonist
b. An extraordinarily potent and irreversible Na-K-ATPase inhibitor
c. An irreversible agonist for angiotensin II receptors
d. Cyanide
e. Nitric oxide
d. Cyanide
- **We have a patient who is diagnosed with variant (vasospastic) angina. Which of the following drugs would be most appropriate, and gen- erally regarded as most effective, for long-term therapy aimed at reducing the incidence or severity of the coronary vasospasm?
a. Aspirin
b. Atorvastatin
c. Diltiazem
d. Nitroglycerin
e. Propranolol
c. Diltiazem
It is generally acceptable and common to administer unfractionated heparin along with other classes of drugs that affect some aspect of the coagulation or thrombolytic processes. The proviso, of course, is to moni- tor closely all drug dosages, the appropriate blood tests, and the patient’s responses overall, since the main risk is uncontrolled or excessive bleeding, if not frank hemorrhage.
There is one main exception. With which one of the following drugs is concomitant administration of heparin contraindicated because of an extremely high risk of excessive bleeding or frank hemorrhage?
a. Alteplase (t-PA)
b. Aspirin
c. Clopidogrel
d. Streptokinase
e. Warfarin
d. Streptokinase
SK not clot specific
A 44-year-old obese man has extremely high plasma triglyceride lev- els, but cholesterol levels are within normal limits. Following treatment with a drug specifically indicated for hypertriglyceridemia, triglyceride levels decrease to almost normal. Which of the following agents is most likely to have caused this desired change?
a. Atorvastatin
b. Cholestyramine
c. Colestipol
d. Ezitemibe
e. Gemfibrozil
e. Gemfibrozil
PPAR activator, stimulates lipoprotein lipase synsthesis